Đến nội dung

Hình ảnh

Topic nhận đề Bất đẳng thức - bài toán tổng hợp


  • Chủ đề bị khóa Chủ đề bị khóa
Chủ đề này có 21 trả lời

#1
E. Galois

E. Galois

    Chú lùn thứ 8

  • Quản lý Toán Phổ thông
  • 3861 Bài viết

Chuyển nhanh đến:
1) Điều lệ
2) Đăng kí thi đấu
3) Lịch thi đấu và tổng hợp kết quả

Topic này dùng để BTC nhận đề thi từ các toán thủ thi đấu.

Điều 3. Phương thức thi đấu, cách tính điểm:
a. Phương thức thi đấu:

- Trước mỗi trận, các toán thủ nộp đề cho BTC, BTC chọn 1 đề thi đấu. Đề thi được chọn là của toán thủ nào thì toán thủ đó gọi là toán thủ ra đề.Toán thủ ra đề không phải làm bài. (BTC đảm bảo nguyên tắc mỗi toán thủ chỉ được chọn đề nhiều nhất 1 lần). Toán thủ đã được chọn đề 1 lần thì những trận sau đó không cần phải nộp đề nữa.
- Trong trường hợp đến hết ngày thứ Tư hàng tuần mà không có toán thủ nào nộp đề, BTC sẽ chỉ định toán thủ có SBD nhỏ nhất (chưa có đề được chọn) phải ra đề.
...

b. Cách tính điểm
...

+ Nếu ra đề sai, đề không đúng chủ đề định sẵn, đề vượt quá cấp học hoặc không giải được đề mình ra, toán thủ ra đề được −30 điểm.
+ Nếu đến lượt mà không ra đề được −20 điểm.
+ Ra đề mà không post đáp án đúng thời gian được −10 điểm
...


Điều 6. Quy định đề bài:
a. Nội dung:
-
Mỗi bộ đề bao gồm 1 câu không copy nguyên văn từ đề thi Olympic quốc gia trở lên.
b. Hình thức:
- Đề bài được gõ Latex rõ ràng.



BTC yêu cầu các toán thủ nộp đề về Bất đẳng thức - bài toán tổng hợp. Đề cần nộp cùng đáp án

Các toán thủ khi thi đấu, cứ yên tâm rằng, sau khi đánh máy là đề đã được lưu, BTC đã nhận được đề của bạn, có điều bạn không nhìn thấy được mà thôi. Bạn nên mừng vì điều này, như thế các toán thủ khác không thể biết trước đề của bạn được.

Bạn cũng nên sử dụng chức năng xem trước của diễn đàn để sửa các lỗi Latex trước khi gửi bài, vì gửi rồi sẽ không xem và sửa lại được nữa.

1) Xem cách đăng bài tại đây
2) Học gõ công thức toán tại: http://diendantoanho...oạn-thảo-latex/
3) Xin đừng đặt tiêu đề gây nhiễu: "Một bài hay", "... đây", "giúp tớ với", "cần gấp", ...
4) Ghé thăm tôi tại 
http://Chúlùnthứ8.vn

5) Xin đừng hỏi bài hay nhờ tôi giải toán. Tôi cực gà.


#2
LuongDucTuanDat

LuongDucTuanDat

    Hạ sĩ

  • Thành viên
  • 59 Bài viết
Tìm bộ số $(x,y,z)$ nguyên dương thỏa mãn
$\left\{\begin{matrix} x^3+y^3+3xyz=z^3 \\ z^3=(2x+2y)^2 \end{matrix}\right.$

Giải:

$\left\{\begin{matrix} x^3+y^3+3xyz=z^3 (1)\\ z^3=(2x+2y)^2(2) \end{matrix}\right.$

Ta có:
$(1) \leftrightarrow x^3+y^3+(-z)^3-3xy(-z)=0$
$\leftrightarrow (x+y-z)(x^2+y^2+z^2-xy+xz+yz)=0$
$\leftrightarrow x+y-z=0$ hoặc $x=y=-z$
Ta có thể loại luôn trường hợp $x=y=-z$ vì $x,y,z$ nguyên dương.
$\rightarrow x+y=z$.
Thay vào (2) ta có:
$z=4(x+y)^2$
$\leftrightarrow z=4=x+y$
Mà $x,y$ nguyên dương nên ta tìm được các bộ số $(x,y,z) \epsilon \begin{Bmatrix} (1,3,4);(3,1,4);(2,2,4) \end{Bmatrix}$

If we only do things that anyone can do it but we just have things that everyone has


#3
caybutbixanh

caybutbixanh

    Trung úy

  • Thành viên
  • 888 Bài viết
Cho $a\geq 2$ Tìm min của:
$$S= a +\frac{1}{a^{2}}$$
Giải
$$S= (\frac{a}{8} +\frac{a}{8} +\frac{1}{a^{2}})+\frac{6a}{8} \geq 3\sqrt[3]{\frac{a}{8}.\frac{a}{8}.\frac{1}{a^{2}}}+\frac{6.2}{8}=\frac{3}{4}+\frac{6}{4}=\frac{9}{4}$$
Min S=$\frac{9}{4}$ <=> a=2

KẺ MẠNH CHƯA CHẮC ĐÃ THẮNG



MÀ KẺ THẮNG MỚI CHÍNH LÀ KẺ MẠNH!.



(FRANZ BECKEN BAUER)




ÔN THI MÔN HÓA HỌC TẠI ĐÂY.


#4
thanhluong

thanhluong

    Trung sĩ

  • Thành viên
  • 122 Bài viết
ĐỀ BÀI CỦA THANHLUONG:

Cho $a$, $b$, $c$ là các số thực dương. Chứng minh rằng
$$\sqrt{a(b+1)}+\sqrt{b(c+1)}+\sqrt{c(a+1)}\leq\frac{3}{2}\sqrt{(a+1)(b+1)(c+1)}$$

ĐÁP ÁN:

Áp dụng bất đẳng thức Bunyakovsky đối với hai bộ số $(\sqrt{a}, 1)$ và $(\sqrt{b+1}, \sqrt{b(c+1)})$, ta được:
$$\left [ \sqrt{a(b+1)}+\sqrt{b(c+1)} \right ]^2=\left [ \sqrt{a} \cdot \sqrt{b+1}+1 \cdot \sqrt{b(c+1)} \right ]^2 \leq (a+1)(bc+2b+1)$$
Ta đưa bài toán về chứng minh:
$$\sqrt{(a+1)(bc+2b+1)}+\sqrt{c(a+1)} \leq \frac{3}{2}\sqrt{(a+1)(b+1)(c+1)}$$


$$\Leftrightarrow \sqrt{bc+2b+1}+\sqrt{c}\leq \frac{3}{2}\sqrt{(b+1)(c+1)}$$.
Tiếp tục sử dụng bất đẳng thức Bunyakovsky:
$$(\sqrt{bc+2b+1}+\sqrt{c})^2=\left (\sqrt{bc+2b+1} \cdot 1+\sqrt{c+1} \cdot \sqrt{\frac{c}{c+1}} \right )^2$$.
$$\leq \left [ (bc+2b+1)+(c+1) \right ] \cdot \left (1+\frac{c}{c+1} \right )=\frac{(b+1)(c+2)(2c+1)}{c+1}$$.


Cuối cùng ta đưa đến chứng minh bất đẳng thức:
$$\sqrt{\frac{(b+1)(c+2)(2c+1)}{c+1}} \leq \frac{3}{2}\sqrt{(b+1)(c+1)}$$.
$$\Leftrightarrow 4(c+2)(2c+1) \leq 9(c+1)^2 \Leftrightarrow 4(2c^2+5c+2) \leq 9(c^2+2c+1)$$.
$\Leftrightarrow c^2-2c+1 \geq 0$.
$\Leftrightarrow (c-1)^2 \geq 0$ (Đúng).
Bài toán được chứng minh hoàn toàn, đẳng thức xảy ra khi và chỉ khi $a=b=c=1$.

Đổi mới là điều tạo ra sự khác biệt giữa người lãnh đạo và kẻ phục tùng.


STEVE JOBS


#5
thanhluong

thanhluong

    Trung sĩ

  • Thành viên
  • 122 Bài viết
Đề bài của thanhluong:
Cho các số thực $a$, $b$, $c$ và $d$ thoả mãn $ad-bc=1$. Chứng minh:
$a^2+b^2+c^2+d^2+ac+bd \geq \sqrt{3}$
Lời giải:
Ta có:
$a^2+c^2+ac = \frac{3}{4}(a+c)^2+\frac{1}{4}(a-c)^2$.
$b^2+d^2+bd = \frac{1}{4}(d-b)^2+\frac{3}{4}(d+b)^2$.
Do đó bất đẳng thức cần chứng minh có thể viết lại thành:
$[3(a+c)^2+(d-b)^2]+[(a-c)^2+3(d+b)^2] \geq 4\sqrt{3}(1)$.
Sử dụng bất đẳng thức Cauchy, ta được:
$3(a+c)^2+(d-b)^2 \geq 2\sqrt{3}(a+c)(d-b)$
$(a-c)^2 + 3(d+b)^2 \geq 2\sqrt{3}(a-c)(d+b)$.
Từ đây suy ra:
$VT_{(1)} \geq 2\sqrt{3}[(a+c)(d-b)+(a-c)(b+d)] = 4\sqrt{3}(ad-bc)=4\sqrt{3}$.
Bài toán được chứng minh hoàn toàn.

Đổi mới là điều tạo ra sự khác biệt giữa người lãnh đạo và kẻ phục tùng.


STEVE JOBS


#6
Dung Dang Do

Dung Dang Do

    Dũng Dang Dở

  • Thành viên
  • 524 Bài viết
http://diendantoanho...bcsqrtca-right/
@@@@@@@@@@@@

#7
phatthemkem

phatthemkem

    Trung úy

  • Thành viên
  • 910 Bài viết
Cho $0<a, b, c, d<1$. Chứng minh rằng có ít nhất một bất đẳng thức sau là sai:

$2a(1-b)>1$

$3b(1-c)>2$

$8c(1-d)>1$

$32d(1-a)>3$


  Hầu hết mọi người đều chấp nhận thua cuộc ngay khi họ sắp thành công. Họ dừng lại

 

ngay trước vạch đích, cách chiến thắng chỉ một bàn chân” -H. Ross Perot

 

  “Tránh xa những kẻ coi nhẹ tham vọng của bạn. Những kẻ nhỏ nhen luôn như thế, còn

 

những người thực sự vĩ đại sẽ khiến bạn cảm thấy rằng bạn cũng có thể trở nên vĩ đại”

 

-Mark Twain

:botay :like :icon10: Huỳnh Tiến Phát ETP :icon10: :like :botay

$WELCOME$ $TO$ $MY$ $FACEBOOK$: https://www.facebook.com/phat.huynhtien.39


#8
junggisung98

junggisung98

    Lính mới

  • Thành viên
  • 7 Bài viết
ra đề ở đâu đay ạ

#9
Khanh 6c Hoang Liet

Khanh 6c Hoang Liet

    Trung sĩ

  • Thành viên
  • 188 Bài viết
Đề của MSS52-Nguyen Viet Khanh 6c :
Cho $n$ số thực $a_{1}, a_{2}, ... ,a_{n}$$,$ $n \geq 2$ thỏa mãn điều kiện $a_{1}a_{2}...a_{n} = 1$. Chứng minh rằng :
$$a_{1}^{2} + a_{2}^{2} + ... + a_{n}^{2} - n \geq \frac{2n}{n - 1}\sqrt[n]{n - 1}\left ( a_{1} + a_{2} + ... + a_{n} - n \right )$$
Hình đã gửi

#10
duaconcuachua98

duaconcuachua98

    Sĩ quan

  • Thành viên
  • 461 Bài viết
Đề của duaconcuachua98

Cho $a,b,c>0$. Chứng minh rằng $\frac{8(a^{2}+b^{2}+c^{2})}{ab+bc+ca}+\frac{27(a+b)(b+c)(c+a)}{(a+b+c)^{3}}\geq 16$

Đáp án:

Áp dụng BĐT Cauchy ta được:
$\frac{8(a^{2}+b^{2}+c^{2})}{ab+bc+ca}+\frac{27(a+b)(b+c)(c+a)}{(a+b+c)^{3}}\geq 2\sqrt{\frac{8(a^{2}+b^{2}+c^{2})}{ab+bc+ca}\cdot\frac{27(a+b)(b+c)(c+a)}{(a+b+c)^{3}} }$
Nên ta cần chứng minh $27(a^{2}+b^{2}+c^{2})(a+b)(b+c)(c+a)\geq 8(ab+bc+ca)(a+b+c)^{3}(1)$
Áp dụng đẳng thức $(a+b)(b+c)(c+a)=(a+b+c)(ab+bc+ca)-abc$ và theo Cauchy: $abc\leq \frac{1}{9}(a+b+c)(ab+bc+ca)$ ta được:
$(a+b)(b+c)(c+a)\geq \frac{8}{9}(a+b+c)(ab+bc+ca)(2)$
Từ $(1)$ và $(2)$ suy ra ta cần chứng minh $3(a^{2}+b^{2}+c^{2})\geq (a+b+c)^{2}\Leftrightarrow (a-b)^{2}+(b-c)^{2}+(c-a)^{2}\geq 0$ (đúng)
Suy ra $dpcm$
Đẳng thức xảy ra khi $a=b=c$

#11
barcavodich

barcavodich

    Sĩ quan

  • Thành viên
  • 449 Bài viết
Cho $a,b,c$ là các số thực thỏa mãn $ \frac{1}{a+b}+\frac{1}{b+c}+\frac{1}{a+c}=1 $ và $ ab+bc+ac >0 $ CMR
\[ a+b+c-\frac{abc}{ab+bc+ac}\ge 4 \]

[topic2=''][/topic2]Music makes life more meaningful


#12
Nguyen Duc Thuan

Nguyen Duc Thuan

    Sĩ quan

  • Thành viên
  • 367 Bài viết
Đề bài: Cho a,b,c>0 thoả mãn: $2a+2b+c\geq 6$ & $a\leq b$. Tìm giá trị nhỏ nhất của biểu thức:
$P=\sqrt{a^2+b^2}+\sqrt{c^2+4b^2}+\sqrt{4(a^2+b^2)+c^2}+a^4+b^3+c^2+b-2c$
Lời giải:
Ta có: $A^2+B^2\geq 2AB$
$\Rightarrow 2(A^2+B^2)\geq (A+B)^2$
Hay $A^2+B^2\geq \frac{(A+B)^2}{2}$ (1) (Dấu "=" xảy ra khi A=B)
Tương tự với 3 số ta cũng được: $A^2+B^2+C^2\geq \frac{(A+B+C)^2}{3}$ (2) ((Dấu "=" xảy ra khi A=B=C)
Áp dụng (1) & (2) Và BĐT Cauchy vào P, ta có:
$P=\sqrt{a^2+b^2}+\sqrt{c^2+(2b)^2}+\sqrt{(2a)^2+(2b)^2+c^2}+(a^4+1+1+1)+(b^3+1+1)+(c^2+4)+b-2c-9$
$\geq \frac{a+b}{\sqrt{2}}+\frac{c+2b}{\sqrt{2}}+\frac{2a+2b+c}{\sqrt{3}}+4a+3b+4c+b-2c-9$
$= \frac{a+3b+c}{\sqrt{2}}+\frac{2a+2b+c}{\sqrt{3}}+2(2a+2b+c)-9$
$\geq 3\sqrt{2}+2\sqrt{3}+12-9=3+3\sqrt{2}+2\sqrt{3}$
(Chú ý rằng: $2a+2b+c\geq 6$ & $a\leq b$ thì suy ra $a+3b+c\geq 6$)
Vậy $Min(P)=3+3\sqrt{2}+2\sqrt{3}$ khi $a=b=1$ & $c=2$ $(\square)$

#13
eatchuoi19999

eatchuoi19999

    Sĩ quan

  • Thành viên
  • 320 Bài viết

Topic này dùng để BTC nhận đề thi từ các toán thủ thi đấu.






BTC yêu cầu các toán thủ nộp đề về Bất đẳng thức - bài toán tổng hợp. Đề cần nộp cùng đáp án

Các toán thủ khi thi đấu, cứ yên tâm rằng, sau khi đánh máy là đề đã được lưu, BTC đã nhận được đề của bạn, có điều bạn không nhìn thấy được mà thôi. Bạn nên mừng vì điều này, như thế các toán thủ khác không thể biết trước đề của bạn được.

Bạn cũng nên sử dụng chức năng xem trước của diễn đàn để sửa các lỗi Latex trước khi gửi bài, vì gửi rồi sẽ không xem và sửa lại được nữa.

Đề bài: Cho $a,b,c$ là các số thực dương thỏa mãn: $ab+bc+ca+abc=4$. Chứng minh rằng:

$a+b+c\geqslant ab+bc+ca$

Lời giải: Đặt $a=\frac{2x}{y+z}$, $b=\frac{2y}{x+z}$, $c=\frac{2z}{x+y}$ với $x,y,z$ là các số thực dương. Khi đó, bất đẳng thức cần chứng minh có thể viết lại thành $\frac{x}{y+z}+\frac{y}{z+x}+\frac{z}{x+y}\geqslant \frac{2xy}{(x+z)(y+z)}+\frac{2yz}{(x+y)(x+z)}+\frac{2xz}{(y+z)(x+y)}$
Áp dụng bất đẳng thức $AM-GM$ ta có:
$VP\leqslant \sum \left [\frac{1}{(x+z)^{2}}+\frac{1}{(y+z)^{2}}\right ]=\sum \frac{xy}{(z+x)^{2}}+\sum \frac{xy} {(z+y)^{2}}=\sum \frac{xz}{(z+y)^{2}}+\sum \frac{xy}{(z+y)^{2}}=\sum \frac{x}{y+z}=VT$
Dấu "$=$" xảy ra $\Leftrightarrow$ $x=y=z$ ($a=b=c=1$)


#14
Khanh 6c Hoang Liet

Khanh 6c Hoang Liet

    Trung sĩ

  • Thành viên
  • 188 Bài viết
Cho $a_1 , a_2 , ... , a_n > 0$. Chứng minh rằng :
$$\frac{a_1}{a_2+a_3} + \frac{a_2}{a_3+a_4} + ... + \frac{a_n}{a_1+a_2} \geq \frac{\left ( a_1+a_2+...+a_n \right )^{2}}{2\left ( a_1^2+a_2^2+...+a_n^2 \right )}$$
Hình đã gửi

#15
4869msnssk

4869msnssk

    Bá tước

  • Thành viên
  • 549 Bài viết

Đề bài: Cho a,b,c là các số dương thoả mãn $a+b+c=6$

Tìm max của $\sqrt{\frac{a+b}{ab+c^{2}}}+\sqrt{\frac{b+c}{bc+a^{2}}}+\sqrt{\frac{a+c}{ac+b^{2}}}$


 B.F.H.Stone


#16
nhatquangsin

nhatquangsin

    Thượng sĩ

  • Thành viên
  • 238 Bài viết

Đề:

Cho: 3ab-a2b-ab2=1

CMR: $\sqrt{\frac{ab}{1+a+a^{2}b}}+\sqrt{\frac{ab}{1+b+ab^{2}}}+\sqrt{\frac{1}{a+b+ab}}\geq \sqrt{3}$

Lời giải:

Ta có:

3ab-a2b-ab2=1$\Leftrightarrow$1+a2b+ab2=3ab$\Leftrightarrow$$\frac{1}{ab}+a+b=3$

Đặt $x=\frac{1}{ab}, y=a, z=b$$\Rightarrow x+y+z=3$

Ta có: $\sqrt{\frac{ab}{1+a+a^{2}b}}+\sqrt{\frac{ab}{1+b+ab^{2}}}+\sqrt{\frac{1}{a+b+ab}}$

=$\frac{1}{\sqrt{x+y+xy}}+\frac{1}{\sqrt{y+z+yz}}+\frac{1}{\sqrt{z+x+zx}}$

Vì: $\frac{1}{\sqrt{x+y+xy}}+\frac{1}{\sqrt{x+y+xy}}+\frac{x+y+xy}{3\sqrt{3}}\geq \frac{3}{\sqrt[3]{3\sqrt{3}}}=\sqrt{3}$

$\Rightarrow 2.\sum \frac{1}{\sqrt{x+y+xy}}+\frac{2(x+y+z)+xy+yz+zx}{3\sqrt{3}}\geq 3\sqrt{3}$

$\Leftrightarrow 2.\sum \frac{1}{\sqrt{x+y+xy}}\geq 3\sqrt{3}-\frac{6+xy+yz+zx}{3\sqrt{3}}$

                                                                    $\geq \frac{6+\frac{(x+y+z)^{2}}{3}}{3\sqrt{3}}=2\sqrt{3}$

$\Rightarrow \sum \frac{1}{\sqrt{x+y+xy}}\geq \sqrt{3}$(đpcm)



#17
4869msnssk

4869msnssk

    Bá tước

  • Thành viên
  • 549 Bài viết

Cho $a,b,c>0$, $ab+bc+ac=3, abc=1$ . CMR:

$\sqrt{\frac{a+b}{ab+c^{2}}}+\sqrt{\frac{c+b}{cb+a^{2}}}+\sqrt{\frac{a+c}{ac+b^{2}}}\leq 3$

Bài giải :

Ta có :

$\sqrt{\frac{a+b}{ab+c^{2}}}\leq \frac{1}{2}\left ( \frac{a+b}{ab+c^{2}}+1 \right )$

$\Rightarrow \sum \sqrt{\frac{a+b}{ab+c^{2}}}\leq \frac{1}{2}\left ( \frac{a+b}{ab+c^{2}}+\frac{b+c}{bc+a^{2}}+\frac{a+c}{ac+b^{2}}+3 \right )$

Vì $a,b,c>0$. Giả sử $a\geq b\geq c>0$

Vì $ab+bc+ac=3, abc=1$ $\Rightarrow \sum \frac{1}{a}=3$

Ta lại có:

$A=\left ( \frac{a+b}{ab+c^{2}}-\frac{1}{c} \right )+\left ( \frac{c+b}{cb+a^{2}}-\frac{1}{a} \right )+\left ( \frac{a+c}{ac+b^{2}}-\frac{1}{b} \right )=\frac{(a-c)(c-b)}{(ab+c^{2})c}+\frac{(a-c)(b-a)}{(bc+a^{2})a}+\frac{(c-b)(b-a)}{(ac+b^{2})b}$

Vì $b^{3}\geq c^{3}\Rightarrow abc+b^{3}\geq abc+c^{3}\Rightarrow \frac{(c-b)(b-a)}{(ac+b^{2})b}\leq \frac{(c-b)(b-a)}{(ab+c^{2})c}$ vì $(c-b)(b-a)\geq 0$

$\Rightarrow A\leq \frac{(a-c)(c-b)}{(ba+c^{2})c}+\frac{(c-b)(b-a)}{(ac+c^{2})c}+\frac{(b-a)(a-c)}{(bc+a^{2})a}=\frac{(b-c)(c-b)}{(ab+c^{2})c}+\frac{(b-a)(a-c)}{(bc+a^{2})a}\leq 0$

$\Rightarrow \sum \sqrt{\frac{a+b}{ab+c^{2}}}\leq \frac{1}{2}\left ( \frac{1}{a}+\frac{1}{b}+\frac{1}{c}+3 \right )=3$

Vậy $\sqrt{\frac{a+b}{ab+c^{2}}}+\sqrt{\frac{c+b}{cb+a^{2}}}+\sqrt{\frac{a+c}{ac+b^{2}}}\leq 3$

Dấu = xảy ra khi $\left\{\begin{matrix} a=b=c & & & & \\ a+b=ab+c^{2} & & & & \\ abc=1 & & & & \\ ab+bc+ac=3& & & & \end{matrix}\right. \Rightarrow a=b=c=1$


 B.F.H.Stone


#18
vuminhhoang

vuminhhoang

    Không Đối Thủ

  • Thành viên
  • 167 Bài viết

cho a,b,c > 0

 

cmr $\dfrac{1}{a+ab}+\dfrac{1}{b+bc}+\dfrac{1}{c+ca} \geq \dfrac{3}{1+abc}$


Mời các mem tham gia

 

100 bài hàm số sưu tầm


#19
duaconcuachua98

duaconcuachua98

    Sĩ quan

  • Thành viên
  • 461 Bài viết

Đề của duaconcuachua98

 

Cho $a,b,c>0$. Chứng minh rằng

$S=\frac{a}{b}+\frac{b}{c}+\frac{c}{a}+\frac{a+b+c}{\sqrt{a^{2}+b^{2}+c^{2}}}\geq 3+\sqrt{3}$

 

Đáp án:

 

Áp dụng BĐT $AM-GM$ ta được:

$\left ( \frac{ab^{2}}{c}+ca\right )+\left ( \frac{bc^{2}}{a}+ab \right )+\left ( \frac{ca^{2}}{b}+bc \right )\geq 2(ab+bc+ca)$

Suy ra $\frac{ab^{2}}{c}+\frac{bc^{2}}{a}+\frac{ca^{2}}{b}\geq ab+bc+ca$

Khi đó ta có:

$(ab+bc+ca)\left ( \frac{a}{b}+\frac{b}{c}+\frac{c}{a} \right )=(a^{2}+b^{2}+c^{2})+(ab+bc+ca)+\left ( \frac{ab^{2}}{c}+\frac{bc^{2}}{a}+\frac{ca^{2}}{b} \right )\geq (a+b+c)^{2}$

Suy ra $\frac{a}{b}+\frac{b}{c}+\frac{c}{a}\geq \frac{(a+b+c)^{2}}{ab+bc+ca}$

Từ đó ta có:

$S\geq \frac{(a+b+c)^{2}}{ab+bc+ca}+\frac{a+b+c}{\sqrt{a^{2}+b^{2}+c^{2}}}=\frac{(a+b+c)^{2}}{ab+bc+ca}+\frac{\sqrt{3}(a+b+c)}{\sqrt{a^{2}+b^{2}+c^{2}}}-\frac{(\sqrt{3}-1)(a+b+c)}{\sqrt{a^{2}+b^{2}+c^{2}}}\geq 2\sqrt[4]{3}.\sqrt{\frac{(a+b+c)^{2}}{(ab+bc+ca)\sqrt{a^{2}+b^{2}+c^{2}}}}-\sqrt{3}(\sqrt{3}-1)=P$

Mặt khác $(ab+bc+ca)^{2}(a^{2}+b^{2}+c^{2})\leq \left [ \frac{2(ab+bc+ca)+(a^{2}+b^{2}+c^{2})}{3} \right ]^{3}=\frac{(a+b+c)^{6}}{27}$

Từ đó suy ra $S\geq P\geq 2\sqrt[4]{3}.\sqrt{3\sqrt{3}}-\sqrt{3}(\sqrt{3}-1)=2\sqrt[4]{81}-(3-\sqrt{3})=3+\sqrt{3}$

Dấu "=" xảy ra khi $a=b=c$



#20
Trang Luong

Trang Luong

    Đại úy

  • Thành viên
  • 1834 Bài viết

Cho $a,b,c,d > 0, a+b+c+d=1$ .Tìm max $S=\left | a-b \right |+\left | a-c \right |+\left | a-d \right |+\left | b-c \right |+\left | b-d \right |+\left | c-d \right |$

 

Đáp án:

Giả sử : $1\geq a\geq b\geq c\geq d$

Ta có : $S=\left | a-b \right |+\left | a-c \right |+\left | a-d \right |+\left | b-c \right |+\left | b-d \right |+\left | c-d \right |\leq a+a+a+b+b+c\leq (a+b+c+d)+(a+b+c+d)+(a+b+c+d)=3$

$Max (S) = 3$ khi : $\left\{\begin{matrix} \left | a-b \right |=a & & & & & & & & \\ \left | a-c \right |=a& & & & & & & & \\ \left | a-d \right |=a & & & & & & & & \\ \left | b-c \right |=b& & & & & & & & \\ \left | b-d \right |=b & & & & & & & & \\ \left | c-d \right |=c & & & & & & & & \\ b+c+d=c+d=d=0 & & & & & & & & \\ a+b+c+d=1 & & & & & & & & \end{matrix}\right. \Leftrightarrow b=c=d=0,a=1$

Và các hoán vị của $a,b,c,d$


"Nếu bạn hỏi một người giỏi trượt băng làm sao để thành công, anh ta sẽ nói với bạn: ngã, đứng dậy là thành công"
Issac Newton




1 người đang xem chủ đề

0 thành viên, 1 khách, 0 thành viên ẩn danh